Proof of Monotone Classes and Subsets in Measure Theory

In summary, the conversation discusses how to show ##\mathcal{F} \subset \sum'##. The proof of hint first shows that ##\sum## is a monotone class, and then uses a similar argument to show that ##\sum'## is also a monotone class. It is then shown that ##\mathcal{F} \subset \sum## and it is needed to show that ##\sum \subset \sum'##. It is proven that ##\mathcal{F} \subseteq \sum'## by showing that if ##N \in m(\mathcal{F})##, then ##A \cap N \in m(\mathcal{F})## for all ##A
  • #1
fishturtle1
394
82
Homework Statement
A family ##\mathcal{M} \subset \mathcal{P}(X)## which contains ##X## and is stable under countable unions of increasing sets and countable intersections of decreasing sets
$$(A_n)_{n\in\mathbb{N}}, A_1 \subset A_2 \subset \dots \subset A_n \subset A_{n+1} \uparrow A = \bigcup_{n=1}^{\infty} A_n \Rightarrow A\in \mathcal{M}$$
$$(B_n)_{n\in\mathbb{N}}, B_1 \supset B_2 \supset \dots \supset B_n \supset B_{n+1} \downarrow B = \bigcap_{n=1}^{\infty}B_n \Rightarrow B \in \mathcal{M}$$
is called a ##\textit{monotone class}##. Assume that ##\mathcal{M}## is a monotone class and ##\mathcal{F} \subset \mathcal{P}(X)## any family of sets.

(iii) If ##\mathcal{F}## is ##\cap##-stable, i.e. ##F, G \in \mathcal{F} \Rightarrow F \cap G \in \mathcal{F}##, then so is ##m(\mathcal{F})##.
[Hint: show that the families
$$\sum := \lbrace M \in m(\mathcal{F}) : M \cap F \in m(\mathcal{F}), \forall F \in \mathcal{F} \rbrace$$
$$\sum' := \lbrace M \in m(\mathcal{F}) : M \cap N \in m(\mathcal{F}), \forall N \in m(\mathcal{F}) \rbrace$$
are again monotone classes satisfying ##\mathcal{F} \in \sum, \sum'##.
Relevant Equations
Here ##m(\mathcal{F})## is the smallest monotone class that contains ##\mathcal{F}##. In other words, if ##L## is a monotone class containing ##\mathcal{F}##, then ##m(\mathcal{F}) \subseteq L##.
My question is how to show ##\mathcal{F} \subset \sum'##. Here is my work for the problem:

Proof of hint: First we'll show ##\sum## is a monotone class. Let ##(A_n)_{n\in\mathbb{N}} \subset \sum## and ##F \in \mathcal{F}##. There are two things to verify. Suppose ##(A_n) \uparrow A = \bigcup_{n=1}^{\infty} A_n##. Then ##(A_n \cap F) \uparrow (A \cap F)##. But ##A_n \cap F \in m(\mathcal{F})## for all ##n##. Hence, ##A \cap F \in m(\mathcal{F})## for all ##F \in \mathcal{F}##. Thus, ##A \in \sum##.

Now, suppose ##(A_n) \downarrow A = \bigcap_{n=1}^{\infty} A_n##. Then ##(A_n \cap F) \downarrow (A \cap F)##. Since ##A_n \cap F \in m(\mathcal{F})## for all ##n##, we have ##A \cap F \in m(\mathcal{F})## for all ##F \in \mathcal{F}##. Thus, ##A \in \sum##. We may conclude ##\sum## is a montone class.

By a similar argument, we can show ##\sum'## is a monotone class. Next, we'd like to show ##\mathcal{F} \subset \sum, \sum'##. Since ##\mathcal{F}## is stable under intersection and ##\mathcal{F} \subset m(\mathcal{F})##, we have ##\mathcal{F} \subset \sum##. I think I need to show ##\sum \subset \sum'## which would give me ##\mathcal{F} \subset m(\mathcal{F}) \subset \sum \subset \sum'##. And clearly ##\sum' \subset m(\mathcal{F})##. This would give ##\sum' = m(\mathcal{F})##, completing the proof.

I tried: Suppose ##A \in \sum##. Let ##N \in m(\mathcal{F})##. We'd like to show ##A \cap N \in m(\mathcal{F})##. If ##N \in \mathcal{F}##, then this is true. Otherwise...
 
  • Like
Likes member 587159
Physics news on Phys.org
  • #2
You have shown that ##\mathcal{F} \subseteq \sum## and ##\sum## is a monotone class. Hence, ##m(\mathcal{F}) \subseteq \sum##.

This allows us to show ##\mathcal{F} \subseteq \sum'##:

If ##A \in \mathcal{F}## and ##N \in m(\mathcal{F})##, then we must show that ##A \cap N \in m(\mathcal{F})##. But now we know that ##N \in \sum##, so since ##A \in \mathcal{F}## we have ##A \cap N \in m(\mathcal{F})##. We conclude ##A \in \sum'##.
 
  • Informative
Likes fishturtle1
  • #3
Math_QED said:
You have shown that ##\mathcal{F} \subseteq \sum## and ##\sum## is a monotone class. Hence, ##m(\mathcal{F}) \subseteq \sum##.

This allows us to show ##\mathcal{F} \subseteq \sum'##:

If ##A \in \mathcal{F}## and ##N \in m(\mathcal{F})##, then we must show that ##A \cap N \in m(\mathcal{F})##. But now we know that ##N \in \sum##, so since ##A \in \mathcal{F}## we have ##A \cap N \in m(\mathcal{F})##. We conclude ##A \in \sum'##.
That makes sense, thank you!
 
  • Like
Likes member 587159

What is the proof of monotone classes and subsets in measure theory?

The proof of monotone classes and subsets in measure theory is a mathematical proof that shows the relationship between monotone classes and subsets in measure theory. It is used to prove the existence of certain measures, such as Lebesgue measure, and to establish important properties of measures.

What is a monotone class in measure theory?

A monotone class in measure theory is a collection of sets that is closed under increasing and decreasing limits. This means that if a sequence of sets in the monotone class is either increasing or decreasing, the limit of the sequence is also in the monotone class.

What is a subset in measure theory?

A subset in measure theory is a set that is contained within another set. In the context of monotone classes, a subset is a set that is included in a larger monotone class.

What is the importance of monotone classes and subsets in measure theory?

Monotone classes and subsets are important in measure theory because they allow us to define and work with measures on more complicated sets. They also help us prove the existence and properties of measures, which are essential in many areas of mathematics and science.

How is the proof of monotone classes and subsets used in practical applications?

The proof of monotone classes and subsets is used in practical applications in a variety of fields, such as probability theory, statistics, and economics. It is also used in the construction of important measures, such as the Lebesgue measure, which is used in many areas of mathematics and physics.

Similar threads

  • Calculus and Beyond Homework Help
Replies
8
Views
1K
  • Calculus and Beyond Homework Help
Replies
2
Views
884
  • Calculus and Beyond Homework Help
Replies
13
Views
967
  • Calculus and Beyond Homework Help
Replies
5
Views
2K
  • Topology and Analysis
Replies
2
Views
154
  • Calculus and Beyond Homework Help
Replies
1
Views
511
  • Calculus and Beyond Homework Help
Replies
2
Views
1K
  • Calculus and Beyond Homework Help
Replies
1
Views
2K
  • Calculus and Beyond Homework Help
Replies
1
Views
266
  • Calculus and Beyond Homework Help
Replies
4
Views
320
Back
Top